3
$\begingroup$

We have a given positive martingale $\rho_t$, with the dynamics: $$\textrm{d} \rho_t = \lambda_t \rho_t \textrm{d} W_t$$ where $W_t$ is a standard Brownian motion. Now we have a "dumped" process p_t: $$\textrm{d} p_t = a_t \lambda_t \rho_t \textrm{d} W_t$$ where $0 \leq a_t \leq 1$.

(Silly) question: are we guaranteed that $p_t$ is also a honest martingale? What would be the easiest way to prove it?

$\endgroup$
1
  • 1
    $\begingroup$ The quick answer is no. You need $\sup\_{s\le t}\rho\_s$ to be integrable to guarantee a positive answer. $\endgroup$ Sep 4, 2011 at 17:08

1 Answer 1

6
$\begingroup$

No, $\rho$ need not be a proper martingale. To guarantee that $p_t=\int_0^ta_sd\rho_s$ is a martingale for all predictable $0\le a_t\le 1$ you need the additional property that $\sup_{s\le t}\rho_s$ is integrable. In fact, for any cadlag martingale $\rho$, the following are equivalent.

  1. $\sup_{s\le t}\vert\rho_s\vert$ is integrable for all $t\in\mathbb{R}^+$.
  2. $\int\xi_sd\rho_s$ is a proper martingale for all bounded predictable processes $\xi$.

The implication (1) ⇒ (2) is a consequence of the Burkholder-Davis-Gundy inequality. This states that there exists positive constants $c < C$ such that $$ c\mathbb{E}\left[[M]_t^{1/2}\right]\le\mathbb{E}\left[\sup_{s\le t}\vert M_s\vert\right]\le C\mathbb{E}\left[[M]_t^{1/2}\right] $$ for all cadlag martingales M with $M_0=0$, where $[\cdot]$ is the quadratic variation (I have written a blog post with a proof of the BDG inequality here). If $M=\int\xi d\rho$ for bounded predictable $\vert\xi\vert\le1$ and (1) is satisfied then $$ \begin{align} \mathbb{E}\left[\sup_{s\le t}\vert M_s\vert\right]&\le C\mathbb{E}\left[\left(\int_0^t\xi^2d[\rho]_s\right)^{1/2}\right]\\\\&\le C\mathbb{E}\left[[\rho]_t^{1/2}\right]\\\\&\le Cc^{-1}\mathbb{E}\left[\sup_{s\le t}\vert\rho_s-\rho_0\vert\right]\\\\& < \infty. \end{align} $$ This is enough to guarantee that the local martingale M is a proper martingale. Conversely, to prove (2) ⇒ (1) then, assuming that $\sup_{s\le t}\rho_s$ is not integrable, you just need to find a bounded predictable process $\xi$ such that $\int_0^t\xi d\rho$ is not a martingale. Supposing that $\rho$ is continuous and writing $\rho^\ast_t=\sup_{s\le t}\rho_s$, you can use the identity $$ \int_0^t f^\prime(\rho^\ast_s)d\rho_s=f(\rho^\ast_t)-f(\rho_0)+f^\prime(\rho^\ast_t)(\rho_t-\rho^\ast_t). $$ This holds for all absolutely continuous functions $f\colon\mathbb{R}\rightarrow\mathbb{R}$. For example, choosing $f^\prime(x)=\sum_{n\in\mathbb{Z}}(-1)^n1_{\{n\le x < n+1\}}$ then $f(x)=\int_0^x f^\prime(y)dy$ will satisfy $0\le f\le1$. So, up to a bounded value, the right hand side of the above identity has absolute value $\vert\rho_t-\rho^\ast_t\vert$, which is not integrable, so is not a martingale. The case where $\rho$ is not continuous can be handled similarly, but is a bit messier.

$\endgroup$
7
  • $\begingroup$ Very interesting, thanks for a complete answer. I did learn quite a lot about stochastic calculus today! I asked another question with some more specific choice for $a_t$. $\endgroup$
    – Grzenio
    Sep 5, 2011 at 9:44
  • $\begingroup$ @ George : Is there a type in your first sentence ? You meant $p$ not $\rho$ right ? PS :I will erase this comment as soon as you corrected your post or (more probably) me. $\endgroup$
    – The Bridge
    Sep 5, 2011 at 11:36
  • $\begingroup$ @The Bridge: No typo. I did mean rho. In fact, the statement about rho implies the same statement about p, by the argument further down in my answer. $\endgroup$ Sep 5, 2011 at 12:42
  • $\begingroup$ @George : Ok, though I thought that by Hypothesis the process $\rho$ was a martingale and not only a local martingale. Am I missing something about the definition of honest or proper martingales ? I thought they were only martingales and that the vocable "proper" or "honest" had no real mathematical meaning. $\endgroup$
    – The Bridge
    Sep 5, 2011 at 13:44
  • $\begingroup$ @The Bridge: yes, "proper" (or honest) martingales are simply martingales, just emphasising the fact that you don't mean local martingales in general. So I don't think you are missing anything there. But, the property that the maximum of rho is integrable is strictly stronger than the martingale property on it's own. $\endgroup$ Sep 5, 2011 at 13:59

Your Answer

By clicking “Post Your Answer”, you agree to our terms of service and acknowledge you have read our privacy policy.

Not the answer you're looking for? Browse other questions tagged or ask your own question.